Anzahl der Möglichkeiten

Neue Frage »

Bujashaka Auf diesen Beitrag antworten »
Anzahl der Möglichkeiten
Folgende Aufgabe bereitet mir Kopfschmerzen bzw. ich werde aus der Fragestellung nicht schlau:

Für soll die Formel



gezeigt werden, indem auf zweierlei Weisen die Anzahl der Möglichkeiten bestimmt wird ( m Frauen und n Männer aus der Menge k Personen )

Wie muss ich das verstehen und wie gehe ich vor?
sqrt(2) Auf diesen Beitrag antworten »

Ganz werde ich aus der Aufgabenstellung nicht schlau, aber vielleicht ist es nur folgendes:

Um k Personen aus n + m auszuwählen, kannst du dies tun, ohne auf das Geschlecht der Personen zu achten. Wie viele Möglichkeiten gibt es?

Um die gleiche Aufgabe zu erledigen, kannst du die Menge in zwei Gruppen mit m und n Personen aufteilen und Personen (mit ) aus der ersten Gruppe auswählen und den Rest aus der anderen Gruppe. Wie viele Möglichkeiten gibt es, auf diese Weise k Personen auszuwählen?
Bujashaka Auf diesen Beitrag antworten »

Nur wie zeigt man in diesem Fall die vorgegebene Formel, so wie es in der Aufgabenstellung steht?

Also hier mal der Originaltext:

Zeigen sie für alle die Formel ... indem Sie auf zweierlei Weisen die Anzahl der Möglichkeiten bestimmen, in einer aus m Frauen und n Männern bestehenden Menschenmenge k Personen auszuwählen.


b) Beweisen sie die Formel durch vollständige Induktion über n.
sqrt(2) Auf diesen Beitrag antworten »

Zitat:
Original von Bujashaka
Nur wie zeigt man in diesem Fall die vorgegebene Formel, so wie es in der Aufgabenstellung steht?

Nun, du berechnest in beidne Fällen, die Möglichkeiten k aus n + m Personen auszuwählen, einmal eben direkt und einmal über den Umweg, zunächst Personen aus einer Gruppe und dann die restlichen aus der anderen auszuwählen. Einmal bekommst du die rechte Seite der Gleichung und einmal die linke Seite. Weil du eben in beiden Fällen das gleiche berechnest, darfst du ein Gleichheitszeichen dazwischen setzen.
AD Auf diesen Beitrag antworten »

Zitat:
Original von Bujashaka
indem auf zweierlei Weisen die Anzahl der Möglichkeiten bestimmt wird ( m Frauen und n Männer aus der Menge k Personen )

Das ist nicht die Aufgabe, sondern eine konkrete Anleitung zum Beweis der Formel! Wie das im Detail geht, hat sqrt(2) geschildert.

Das ist übrigens das in der Kombinatorik häufig genutzte Prinzip des doppelten Abzählens.


P.S.: Dieses Prinzip hat mir damals zu einer sehr einfachen Lösung dieser IMO-Aufgabe (A1) verholfen. Augenzwinkern
Bujashaka Auf diesen Beitrag antworten »

Ok, was sqrt geschrieben hat, ist sehr einleuchtend und schriftlich gut umzusetzen. Nur ist es mit einer Erklärung getan oder soll diese Aussage durch eine Rechnung belegt werden?

b)

Bei dem b - Teil, habe ich beim Induktionsanfang ein Problem. Die Induktion soll über n gehen. Beim IA wird n = 0 gesetzt, doch was geschieht mit den anderen Faktoren in der Formel? Laut der Summe soll ja j = 0 sein, aber was geschieht mit k?
 
 
sqrt(2) Auf diesen Beitrag antworten »

Zitat:
Original von Bujashaka
Nur ist es mit einer Erklärung getan oder soll diese Aussage durch eine Rechnung belegt werden?

Du könntest begründen, wie das Auswählen durch den Binomialkoeffizienten repräsentiert wird und wie die einzelnen Bestandteile der linken Seite der Gleichung zu Stande kommen.

Zitat:
Original von Bujashaka
Beim IA wird n = 0 gesetzt, doch was geschieht mit den anderen Faktoren in der Formel? Laut der Summe soll ja j = 0 sein, aber was geschieht mit k?

Wenn die Induktion über n geht, wird k (wie auch m) als fest angenommen.
Bujashaka Auf diesen Beitrag antworten »

Die Begründungen für den ersten Teil habe ich.

Bei der Induktion verstehe ich nicht, wie es mit dem "k (m) als fest angenommen" umzusetzen ist?
sqrt(2) Auf diesen Beitrag antworten »

Das heißt eigentlich nicht viel anderes, als dass du k und m einfach in Ruhe lässt. Du behandelst sie als Konstanten, deren Wert du nicht kennst, so wie du es ständig tust, wenn du Terme mit Unbekannten umformst.
AD Auf diesen Beitrag antworten »

In b) sollst du die Formel



durch Induktion über zeigen.

Den Induktionsanfang solltest du selbst hinkriegen.

Im Induktionsschritt kannst du die dem Pascalschen Dreieck zugrundeliegende Formel



nutzen, rechts übrigens auch:

Bujashaka Auf diesen Beitrag antworten »

Ok, dankeschön. Freude

Wenn ich n=0 setze, ist mir unklar gewesen, gleich was ich k und m setzen soll.
Bujashaka Auf diesen Beitrag antworten »



Im Induktionsanfang kriege ich für eine wahre Aussage heraus ( 1 = 1 )

Im Indukionsschritt geht es so weiter :



IV:

=

Ist damit die VI abgeschlossen oder habe ich da wieder etwas falsch gemacht? Hab es nach meinen Vorlagen versucht.
sqrt(2) Auf diesen Beitrag antworten »

Zitat:
Original von Bujashaka
Im Indukionsschritt geht es so weiter :


Du musst das n schon überall ersetzen:



Mit Arthurs Gleichungen oben und einmaliger Verwendung der IV kommst du auf



Bedenke jetzt noch, dass

,

dann kannst du nach einer kleinen Umbenennung die IV noch einmal anwenden und bist dann schon fertig.
Bujashaka Auf diesen Beitrag antworten »

Jetzt bin ich irgendwie ganz aus dem Konzept.



IV:

= , da


Oh man, ich glaub jetzt bin ich voll auf dem Holzweg... traurig
sqrt(2) Auf diesen Beitrag antworten »

Zitat:
Original von Bujashaka

Das ist so weit richtig.

Zitat:
Original von Bujashaka
IV:

=

Das ist falsch und was hat das mit der IV zu tun?

Zitat:
Original von Bujashaka

Bestimmt nicht.

Was machst du da eigentlich?



Jetzt kannst du die IV anwenden, die da lautet

.
Bujashaka Auf diesen Beitrag antworten »

Ja und genau dort scheitert es ja bei mir...

Wenn ich die IV anwende gehe ich so weiter vor:

IV:
Bujashaka Auf diesen Beitrag antworten »

So wie ich es von der vollständigen Induktion kenne, habe ich es auch auf diese Aufgabe angewendet...



...wobei ich leztendlich die IV rauskriegen möchte.

Da dies anscheinend der falsche Schritt zur Lösung ist, kannst du mir das weitere Vorgehen erläutern?

Gott
sqrt(2) Auf diesen Beitrag antworten »

Zitat:
Original von Bujashaka
So wie ich es von der vollständigen Induktion kenne, habe ich es auch auf diese Aufgabe angewendet...


Was willst du mir mit diesem Term eigentlich sagen? Gleichungen sind hier das einzig Wahre...

Zitat:
Original von Bujashaka
...wobei ich leztendlich die IV rauskriegen möchte.

Das funktioniert, wenn man die IV nur einmal verwenden muss (hier nicht der Fall) und führt auch nur zu richtigen Ergebnissen, wenn man auf äquivalente Umformungen achtet.

Zitat:
Original von Bujashaka
Da dies anscheinend der falsche Schritt zur Lösung ist, kannst du mir das weitere Vorgehen erläutern?

"Die IV anwenden" heißt, wenn die IV die Form hat: Wenn in deinem Induktionsschritt irgendwo vorkommt, darfst du es durch ersetzen, denn es wird ja angenommen, dass .

(IV bei mir: Induktionsvoraussetzung, auch genannt Induktionsannahme, nicht Induktionsverankerung (gleich Induktionsanfang))
Neue Frage »
Antworten »



Verwandte Themen

Die Beliebtesten »
Die Größten »
Die Neuesten »